Tải bản đầy đủ (.pdf) (31 trang)

Tài liệu Bất đẳng thức AM-GM doc

Bạn đang xem bản rút gọn của tài liệu. Xem và tải ngay bản đầy đủ của tài liệu tại đây (7.3 MB, 31 trang )


1





Lời nói đầu.

Bất đẳng thức là một lĩnh vực khá hay và khó đối với mỗi chúng ta. Hiện này có
khá nhiều người quan tâm đến nó bởi vì nó thực sư rất đơn giản, quyến rũ và bạn không
cần phải “học vẹt” nhiều định lý để có thể giải được chúng. Mỗi người trong chúng ta,
đặc biệt là các bạn yêu toán, dù ít hay nhiều thì cũng đã từng đau đầu trước một bất đẳng
thức khó và cũng đã từng có được một cảm giác tự hào khi mà mình chứng minh được bất
đẳng thức đó.Không biết các bạn nghĩ thế nào nhưng theo quan điểm của chúng tôi, thì
nếu ta học tốt bất đẳng thức thì cũng có thể học tốt các lĩnh vực khác của toán học vì như
đã nói, bất đẳng thức đòi hỏi chúng ta phải có một kiến thức tổng hợp tương đối vững
vàng.Có lẽ nhiều bạn không tin nhưng chắc hạn bạn cũng biết đến anh Phạm Kim Hùng,
sinh viên hệ CNTN khoa toán, trường ĐHKHTN, ĐHQG Hà Nội, người đã từng tham dự
hai kì thi IMO và đều đoạt kết quả cao nhất trong đội tuyển Việt Nam. Trong thời học phổ
thông, anh ấy chỉ chuyên tâm rèn luyện bất đẳng thức thôi.
Có thể nói hiện nay có rất nhiều phương pháp hiện đại chẳng hạn như SOS; hay
dồn biến, hay EV…. Để chứng minh bất đẳng thức nếu sử dụng chúng thì hầu như bài nào
cũng giải được. Thời gian để giải theo cách đó cũng khá tốn. Bởi vậy, vấn đề tìm ra lời
giải theo các cách cổ điển luôn được đánh giá cao. Trong bài viết này, chúng tôi xin giới
thiệu về một bất đẳng thức kinh điển là AM-GM
Trong bài viết còn nhiều thiếu sót. Mọi thắc mắc xin liên hệ email:

Người viết: Phạm Tiến Giang.



2

I) Bất đẳng thức AM-GM là gì ?
Bất đẳng thức AM-GM được 3 nhà toán học Schwar, Bunhiacopxki và Cauchy
phát minh ra. Tuy nhiên, mọi người vẫn thường gọi đây là bất đẳng thức Cauchy nhiều
hơn và cũng thường nhầm lẫn rằng Cauchy tìm ra bất đẳng thức này. Có lẽ do ông là
người đã đưa ra cách chứng minh bất đẳng thức AM-GM rất hay và độc đáo.
Bất đẳng thức AM-GM là tên chuẩn quốc tế được viết tắt từ Arithmetic Means-
Geometric Means.
Trước hết, chúng tôi xin được giới thiệu bất đẳng thức AM-GM được phát biểu
như sau:
Với n số không âm a
1
, a
2
, a
3
,…
,
a
n
, ta luôn có:
1 2
1 2

.
n
n
n
a a a

a a a
n
  

Dấu “=” xảy ra khi: a
1
= a
2
= a
3
=…= a
n


Bất đẳng thức này có hơn 20 cách chứng minh. Trong đó, cách chứng minh bằng
quy nạp bất không tuần tự(hay còn gọi là quy nạp kiểu Cauchy ) được coi là hay nhất.
Ý tưởng: chúng ta chứng minh qua 3 bước cơ bản sau:
Bước 1: Chứng minh bất đẳng thức đúng với n = 2.
Bước 2: Giả thiết rằng bất đẳng thức đúng với n,trên cơ sở đó chứng minh bất
đẳng thức đúng với 2n.
(Nhận xét: với kết luận ở bước 1 và kết quả ở bước 2, ta thấy bất đẳng thức đúng
với n = 2, 4, 8, 16,…)
Bước 3: Giả thiết rằng bất đẳng thức đã đúng với n (n > 2),trên cơ sở đó chứng
minh bất đẳng thức đúng với n – 1.
(Nhận xét: sau bước 2, ta biết bất đẳng thức đúng với mọi n = 2
k
(k tùy ý), thì ta
bước 3 cho hay bất đẳng thức đúng với mọi
2
k

n

, mà k tùy ý, nên bất đẳng thức đúng
với mọi số tự nhiên
2
n

.)





3

Chứng minh:
*) Với n = 2, bất đẳng thức ở dang:

.
2
a b
a b



; 0
a b
 
(1)
Khi đó:

 
2
2 2
(1) 2 4
4
a b
ab a b ab ab

     


 
2
2 2
2 0 0
a b ab a b
      
hiển nhiên đúng.
Dấu “=” xảy ra khi : a = b
*) Giả sử bất đẳng thức đúng với n số không âm. Xét 2n số không âm. a
1
, a
2
, a
3
,…
,
a
n
, a

n+1
,…
,
a
2n
,
Ta có:
 
1 2 1 2 2
1 2 2

1 1

2 2
n n n n
n
a a a a a a
a a a
n n n
 
     
 
    
 
 



1 2 1 2 2 1 2 1 2 2
1

. . . . .
2
n n n n
n n n n n n n n
a a a a a a a a a a a a
   
  

2
1 2 1 2
. .
n
n n n
a a a a a



Dấu “=” xảy ra khi:
1 2
1 2 2 1 2 1 2
1 2 1 2 2


. .
n
n n n n n n
n n
a a a
a a a a a a a a
a a a a a a

  
  


        





*) Giả sử bất đẳng thức đã đúng cho n số không âm. Lấy n – 1 số không âm: a
1
, a
2
,
a
3
,…
,
a
n-1
. Đặt

1 2 1

1
n
n
a a a
a

n

 





0
a
 

Ta có:
1 2 1 1 2 1
1 2 1 1 2 1
1
. .
1 1
n n
n
n n
a a a a a a
a a a a a a
n n n
 
 
   
   
   
   

 
   

hay
1 2 1 1 2 1
1 2 1

. .
1 1
n n
n
n
a a a a a a
a a a
n n
 

   
 

 
 
 


4

Nâng cả hai vế lên lũy thừa bậc n, ta được:
1 2 1 1 2 1
1 2 1


. .
1 1
n
n n
n
a a a a a a
a a a
n n
 

   
   

   
 
   

Vì chỉ cần xét trường hợp a
1
+ a
2
+ a
3
+…+

a
n-1
> 0, nên suy ra:
1

1 2 1
1 2 1

.
1
n
n
n
a a a
a a a
n



 
 

 

 

Lấy căn bậc n – 1 của hai vế, thì được:
1 2 1
1
1 2 1

.
1
n
n

n
a a a
a a a
n



 
 


Hiển nhiên dấu “=” xảy ra khi:
1 2 1
1 2 1 1 2 1


1
n
n n
a a a
a a a a a a
n

 
 
     


Ghi chú: Ở đoạn này, ta có thẻ lùi từ 2n xuống n + 1.
Ví dụ 1.Chứng minh rằng với mọi số thực dương a, b, c ta có:


1 1 1 9
a b c a b c
  
 

Lời giải: Sử dụng bất đẳng thức AM-GM cho 3 số, ta có:

 
3
3
1 1 1 3
3 9
a b c abc
a b c
abc
 
     
 
 

Bất đẳng thức được chứng minh. Dấu bằng xảy ra khi a = b = c.
Ví dụ 2.(Bất đẳng thức Nesbitt) Chứng minh rằng với mọi số thực dương a, b, c ta có:

3
2
a b c
b c c a a b
  
  


Lời giải: Đây là rất quen thuộc và có rất nhiều cách chứng minh.
Cách 1. Áp dụng kết quả của ví dụ 1 , ta được:
1 1 1 3
a b c a b c
b c c a a b b c c a a b
     
        
     
     
     

 
1 1 1
3
a b c
b c c a a b
 
     
 
  
 


5

     
1 1 1 1
3
2

a b b c c a
b c c a a b
 
        
 
 
 
  
 

1 3
.9 3
2 2
  
.
Chứng minh hoàn tất. Dấu bằng xảy ra khi a = b = c.

Cách 2. Xét biểu thức sau:

a b c
S
b c c a a b
  
  


b c a
M
b c c a a b
  

  


c a b
N
b c c a a b
  
  

Ta có: M + N = 3. Mặt khác, theo bất đẳng thức AM-GM thì

3
a b b c c a
M S
b c c a a b
  
    
  


3
a c a b b c
N S
b c c a a b
  
    
  

Vậy
2 3 2 3

M N S S
    
. Bất đẳng thức được chứng minh.
Dấu bằng xảy ra khi a = b = c.
Đối với bất đẳng thức Nesbitt cho 4 số thì chứng minh tương tự như các này.
Sau đây là một số kĩ thuật đơn giản nhưng cũng thường được sử dụng để áp dụng
bất đẳng thức AM-GM cùng với đó một số bất đẳng thức liên quan tới AM-GM.


II) Kĩ thuật chọn điểm rơi:
Bất đẳng thức AM-GM là một bất đẳng thức thuần nhất. Vì thế, chúng rất hữu hiệu
trong việc chứng minh các bất đẳng thức thuần nhất. Tuy nhiên, do điều kiện xảy ra dấu
bằng của các bất đẳng thức này rất nghiêm ngặt nên việc áp dụng một cách trực tiếp và

6

máy móc đôi khi khó đem lại kết quả. Để áp dụng tốt các bất đẳng thức này, chúng ta phải
nghiên cứu kỹ điều kiện xảy ra dấu bằng và áp dụng kĩ thuật chọn điểm rơi này.
.Chúng ta hãy xét một số ví dụ sau:
Ví dụ 1: Cho . Tìm GTNN của :
Lời giải


Cộng 2 bất đẳng thức trên, ta có

. Dấu "=" xảy ra khi và chỉ khi
*)Tuy nhiên vấn đề đặt ra là tại sao nghĩ ra được số để thêm vào bất đẳng thức?
Để giải quyết vấn đề này, sử dụng ý tưởng dùng bất đẳng thức như trên, nhưng ta sẽ thêm
vào 1 số nào đó:



Cộng hai bất đẳng thức trên ta có:

Dấu "=" xảy ra khi và chỉ khi:
Giả sử đã tồn tại để dấu "=" xảy ra, khi đó


Thay vào F được GTNN của F là đạt được khi .
Như vậy việc đưa số vào áp dụng BĐT là hoàn toàn có cơ sở. Từ đó ta nâng bài
toán lên với hệ số các số hạng là các số dương như sau:
Ví dụ 2: Cho .
Tìm GTNN của

7

Lời giải:
Mục tiêu của chúng ta là dùng bất đẳng thức AM-GM sao cho khi cộng 2 bất đẳng
thức vào, ta có vế trái là 2F cộng với 1 số hạng nào đó, còn vế phải chứa biểu thức đã cho
trong giả thiết. Rõ ràng việc đặt số đơn lẻ sẽ không đưa đến kết quả mà phải biến đổi
số hạng cộng vào mỗi bất đẳng thức.


Cách đặt số hạng cộng vào này giúp ta triệt tiêu được c bên vế trái, nhân thêm
được hệ số a vào vế phải. Ta tiếp tục cộng 2 bất đẳng thức :

Dấu "=" xảy ra khi và chỉ khi
.
Khi đó . Giả sử đã có

thỏa mãn dấu "=", tức là:

(1)


Khi đó theo (1) tìm được GTNN của F là

Lần này, ta sẽ phát triển bài toán theo hướng tăng dần số mũ. Để tránh phức tạp, ta
cho các hệ số bằng 1.
Ví dụ 3: Cho . Tìm GTNN của
Lời giải
Áp dụng bất đẳng thức AM-GM cho 4 số dương:
;

8

Ở đây, ta cộng 3 số hạng bậc 4 của x với 1 số hạng tự do. Mục đích là để khi ta áp
dụng BĐT AM-GM, ta thu được một số hạng bậc 3 của x.
Cộng 2 bất đẳng thức, ta được :
.
Dấu "=" xảy ra khi và chỉ khi: .
Khi đó (2). Giả sử tồn tại để dấu bằng xảy ra, vậy thì:


.
Thay vào (2) ta có , đạt được khi x = y = .
Qua các ví dụ trên, có lẽ các bạn đã phần nào hiểu được kĩ thuật này. Tuy nhiên,
các ví dụ này vẫn còn rất đơn giản và mang tính chất tương tự nhau. Chúng ta hãy xét một
trường hợp khác với các ví dụ trên xem thế nào?
Ví dụ 4. Chứng minh rằng nếu x, y, z là các số thực không âm thì

 

   
3
2 2 2 2 2 2
2
6 27 10
x y z x y z xyz x y z
        

Lời giải:
Sử dung nguyên lý cơ bản “dấu bằng xảy ra khi một cặp biến số nào đó bằng
nhau”, ta có thể tìm được dấu bằng của bất đẳng thức trên xảy ra khi y = z = 2x. Điều này
cho phép chúng ta mạnh dạn đánh giá như sau:

 
 
 
3
2 2 2 2 2 2
2
10 6
VP VT x y z x y z x y z
         


   
 
1
2 2 2 2 2 2
2
10 6

x y z x y z x y z
 
        
 
 


     
 
1 1
2 2 2 2 2 2 2 2 2
2 2
10
. 1 2 2 6
3
x y z x y z x y z
 
          
 
 


 
   
2 2 2
10
. 2 2 6
3
x y z x y z x y z
 

        
 
 


9






2 2 2
28 2 2
3
x y z x y z
   

(1)
Áp dụng bất đẳng thức AM-GM, ta có:

4 4
2 2 2 2 2 8 8
2 2 2 2 2
9
9
8
4 4 9 9
4 4 4 4 4
y z y z x y z

x y z x x
       
      
       
       


8 7
9
28 2 2 7.4 2 2 9 4
x y z x y z x yz
     

Nhân hai bất đẳng thức trên vế theo vế, ta được:

 
 
2 8 8
2 2 2 8 7
9
9
8
28 2 2 9 4 .9 81
4
x y z
x y z x y z x yz xyz
     
(2)
Từ (1) và (2), ta suy ra bất đẳng thức cần chứng minh.
Trong ví dụ này, chúng ta đã sử dụng bất đẳng thức AM-GM cùng với kĩ thuật

chọn điểm rơi. Lời giải rất hiệu quả và ấn tượng. Tuy nhiên sự thành công của lời giải trên
nằm ở ba dòng ngắn ngủi ở đầu. Không có được “dự đoán” đó, khó có thể thu được kết
quả mong muốn.
Chúng ta hãy xét tiếp một ví dụ khác:
Ví dụ 5. Cho a;b;c>0. Chứng minh rằng :

     
3 3 3
3 3 3
3 3 3
1
a b c
a b c b a c c b a
  
     


Ý tưởng: Đây là một bất đẳng thức hay và khó.Chúng ta có khá nhiều cách giải bài
này, ví dụ như :
Sử dụng bất đẳng thức AM-GM, ta có:

 
 




2
2
3 2

1 1
1 1 1 1
2 2
x x x
x
x x x x
   
       

Vì vậy, ta có:
 
3
3 2 2 2
3
3
2
1 1 1
1
1
1
1
1
2
cyc cyc cyc cyc
a
c b
a b c b c
b c
a
a

a
   

  
 

 



 
 
 
 
   


10

Tuy nhiên, ở đây, chúng tôi xin giới thiệu 1 cách giải khác khá độc đáo và có sử
dụng kĩ thuật chọn điểm rơi như sau:
Nhận xét rằng dấu bằng xảy ra khi và chỉ khi a = b = c.
Ta sẽ tìm p sao cho bất đẳng thức sau đúng:
 
3
3
3
p
p p p
a a

a b c
a b c

 
 

Thay a = b = c vào bất đẳng thức trên, ta suy ra được p = 2.
Vậy, vấn đề của ta bây giờ là chứng minh bất đẳng thức sau đúng.

 
3 2
3
2 2 2
3
a a
a b c
a b c

 
 

Từ những dự đoán và suy luận như trên, ta có lời giải bài toán trên như sau:
Lời giải:
Ta sẽ chứng minh :
 
3 2
3
2 2 2
3
a a

a b c
a b c

 
 
(*)
Thật vậy:
 
2 2 2
3
3
1
(*)
a
a b c
a b c
 
 
 




 


2
3
2 2 2 3
a b c a a b c

     






 
2
3
2 2 2 2 2
2
a b c b c a b c
     










 
3
2 2 2 2 2 2
b c a b a c a b c
      





   




   
2 2 3
2 2 2 2
2
b c a b a c b c a b c a b c
         




   


  
2 2 2
2 2
0
b c a b a c a b c b c
        
(Đúng)
Vậy (*) đúng. Chứng minh tương tự, ta có:


 
3 2
3
2 2 2
3
b b
a b c
b a c

 
 


11


 
3 2
3
2 2 2
3
c c
a b c
c a b

 
 

Do đó
     

3 3 3 2 2 2
3 3 3
2 2 2
3 3 3
1
a b c a b c
a b c
a b c b a c c b a
 
   
 
     

Vậy, ta suy ra bất đẳng thức cần chứng minh.

Sau đây là một số bài tập nho nhỏ về kĩ thuật này:
1) cho
1
0;
2
a
 



 
Tìm min
2
1
a

a


2) cho


;
a m n


0
m n
 

;m n


. Tìm min
1
n
m
a
a


3) Cho
3
; ;
4
a b c

 
1
a b c
  

Chứng minh:
2 2 2
9
1 1 1 10
a b c
a b c
  
  

4) Cho
; ;a b c



;
3
a b c
  

Chứng minh:
a)
5 5 5 5
2 2 2 3 3
a b b c c a     


b)
5
5 5 5
( )(2 ) ( )(2 ) ( )(2 ) 3 6
a a c a b b b a b c c c b c a        

5) cho
; ;a b c



thỏa mãn
3
a b c
  
. CMR:
3 4 5
1
a b c


6)
0
a b
 
CMR:
2
32
2 5
( )(2 3)

a
a b b
 
 

7) Cho
4
a b
 
CMR:
6 10
2 3 18
a b
a b
   

8) Cho


; ; 2;2
a b c 

3
a b c
  
, CMR:
2 2 2
4 4 4 3 3
a b c     
9) Cho

; ;x y z



CMR:
4
1
(1 3 )( 8 )( 9 )( 6) 7
xyz
x x y y z z

   


12

III) Kĩ thuật Cô-si ngược dấu:
Các bạn cứ tưởng tượng rằng khi ta dùng bất đẳng thức AM-GM rồi nghịch đảo
lại, rồi lại đặt dấu trừ ở trước thì hiển nhiên bất đẳng thức sẽ không đổi chiều.
Sau đây là một số ví dụ để các bạn nắm rõ kĩ thuật này và một số bài để tự luyện.
Các bài toán mẫu:
Bài toán 1. Cho các số dương a,b,c thỏa mãn a+b+c=3. Chứng minh

Lời giải:
Ta có :
Tương tự cho 2 bất đẳng thức kia, ta suy ra:

Bài toán 2. Cho các số thực dương a,b,c có tổng bằng 3. Chứng minh

Lời giải: ta có:


Tương tự cho 2 bất đẳng thức còn lại ta suy ra


Một số bài tập tự luyện:
Bài 1. Cho các số dương a,b,c,d thỏa mãn a+b+c+d=4. Chứng minh
+
Bài 2. Cho các số dương a,b,c,d thỏa mãn a+b+c+d=4. Chứng minh

Bài 3 . Chứng minh rằng với mọi số thực dương ta luôn có:


13

Bài 4. Chứng minh rằng với mọi số thực dương ta luôn có:

Bài 5. Chứng minh rằng với mọi số thực không âm a,b,c
sao cho a+b+c=3 thì

Bài 6. Cho các số thực dương a,b,c có tổng bằng 4. Chứng minh

Bài 7. Với mỗi số thực dương a,b,c,d có tổng bằng 4.
Chứng minh rằng :

Tóm lại kĩ thuật Cauchy ngược dấu rất có lợi trong các bất đẳng thức hoán vị

IV) Bất đẳng thức Cauchy-Schwarz:
Đã nhắc đến các bất đẳng thức cổ điển, đặc biệt là bất đẳng thức AM-GM, chúng
không thể không nhắc đến bất đẳng thức Cauchy-Schwar. Đây là bất đẳng thức cũng rất
quen thuộc với các bạn học sinh phổ thông và việc nắm chắc sử dụng thành thạo bất đẳng

thức này là rất cần thiết cho tất cả bạn đọc, không chỉ với các bạn thi học sinh giỏi mà
ngay cả với các bạn ôn thi vào Đại học.
Trước hết xin nhắc lại về bất đẳng thức này, nó được phát biểu như sau:




 
2
2 2 2 2 2 2
1 2 1 2 1 1 2 2
. . .
n n n n
a a a b b b a b a b a b
         

Dấu “=” xảy ra khi và chỉ khi:
i j j i
a b a b

với mọi
i j


Chứng minh: Có lẽ các bạn ai cũng biết cách chứng minh bất đẳng thức này nên
chúng tôi chỉ xin nêu 1 cách chứng minh đơn giản và tiêu biểu sau :
Xét tam thức bậc 2 sau đậy:
       
2 2 2
1 1 2 2


n n
f x a x b a x b a x b
      

Sau khi khai triển ta có:

14









2 2 2 2 2 2 2
1 2 1 1 2 2 1 2
2
n n n n
f x a a a x a b a b a b x b b b
           

Mặt khác, vì


0f x x
  


nên theo định lý về tam thức bậc 2.




 
2
2 2 2 2 2 2
1 2 1 2 1 1 2 2
0
f n n n n
a a a b b b a b a b a b
            

Đẳng thức xảy ra khi phương trình


0
f x

có nghiệm, nói cách khác
i j j i
a b a b

với mọi
i j

.
Ta có 1 dạng khác của bất đẳng thức này và cũng rất hay được sử dụng và nó được
phát biểu như sau:

Với 2 bộ số


1 2
; ; ;
n
a a a



1 2
; ; ;
n
b b b
thỏa mãn b
i
dương, ta có:
 
2
22 2
1 2
1 2
2 2 2
1 2 1 2



n
n
n n

a a a
a
a a
b b b b b b
  
   
  

Dấu “=” cũng xảy ra khi và chỉ khi
i j j i
a b a b

với mọi
i j


( Lưu ý là để sử dụng tốt dạng, các bạn cần có cái nhìn “ hai chiều ”)

Sau đây là một số ví dụ cơ bản:
Ví dụ 1. cho a;b;c là 3 số thực dương tùy ý. Chứng minh rằng:
2 2 2 4
bc ac ab a b c
b c a a c b b a c
 
  
     

Lời giải: Sử dụng bất đẳng thức Cauchy-Schwarz, ta có:
2 2 2
1 1 (1 1)

( ) ( )
a b a c a b a c

 
    

Suy ra:
1 1 1 1
2 4
a b c a b a c
 
 
 
   
 

Theo đánh giá này, ta được:

1
2 4 4
bc bc bc a b c
a b c a b a c
 
 
  
 
   
 
 



15

Đó chính là điều phải chứng minh.
Nhận xét: Lời giải tuy đơn giản nhưng để tìm được hướng làm thì không dễ chút
nào. Điểm đặc biệt ở đây là việc phát hiện ra hằng đẳng thức:
bc bc
a b c
a b a c
 
   
 
 
 


Từ đó, chúng ta có cách tách thích hợp để có thể sử dụng bất đẳng thức Cauchy-
Schwarz như trên.

Ví dụ 2. cho a, b, c là các số dương có tổng bằng 3. Chứng minh rằng:
2 2 2 2 2 2 2 2 2
1 1 1 1
4 4 4 2
a b c b a c c b a
  
     

Gợi ý lời giải: Sử dụng ý tưởng như trên, ta sẽ tìm một hằng đẳng thức thích hợp
để có thể giúp ta chứng minh bất đẳng thức đã cho. Chúng ta hãy thử để ý đến hằng đẳng
thức sau :

2 2
2 2 2 2
3
b c
a b a c
 
 
 
 
 


Từ đó, ta nghĩ đến cách tách




2 2 2 2 2 2 2 2
4 2
a b c a a b a c
      
và sử
dụng bất đẳng thức Cauchy-Schwarz như sau:
 
2
2 2 2
2 2 2 2 2 2 2 2
4 2
a b c
a b c

a b c a a b a c
 
  
   

Theo đánh giá này, ta có:
2 2 2
2 2 2 2 2 2 2 2
1 9
4 2 2
a b c
a b c a a b a c
 
   
 
   
 
  

Chia cả 2 vế cho 9, ta được bất đẳng thức cần chứng minh.

Ví dụ 3. Chứng minh rằng nếu a, b, c, d là các số thực dương và
4
1
r abcd
 
, bất đẳng
thức sau luôn đúng:

16




2
4 1
1 1 1 1
1 1 1 1 1
r
ab bc cd ad
a b c d r

   
   
    

Lời giải: Chú ý rằng giả thiết
4
1
r abcd
 
ta suy ra tồn tại các số thực dương x;
y; z; t sao cho:
, , .
ry rz rt rx
a b c d
x y z t
   

Khi đó, bất đẳng thức được viết lại dưới dạng sau:
 

2
2 2
2
2
1
1 1
1
4 1
1
1 1 1 1
r t
r z r yr x
r
y
x t
z
ry rx rt rx
r
x y z t

 


   

   



2

2 2 2 2
4 1
1
r
r z x r t y r x z r y t
ry x rx y rt z rx t r

   
    
    

Xét 2 biểu thức
x z y t x z y t
A
ry x rx y rt z rx t
   
   
   
;
z t x y
B
ry x rx y rt z rx t
   
   

Ta phải chứng minh

 



2
2
4 1
1
1
r
A r B
r

  


Theo bất đẳng thức AM-GM dễ thấy




4 8
r xy yz zt tx xz yt
    













4 1 4 8 8
r x z y t x z y t xz yt
        


       
2 2 2
1 2 1
r x y z t x y z t r x y z t
             

Sử dung bất đẳng thức Cauchy-Schwarz, chú ý rằng
1
r

ta có:

   
1 1 1 1
A x z y t
ry x rt z rx y rz t
   
     
   
   
   



17

     
      
2
2 2
4 4 4
2
x z y t x y z t
x z ry rt y t rx rz
x z y t r x z y t
    
  
     
     

8
1
r




 
       
2
x y z t
B
z xy x t rz y x rt z y rx t
  


      


 
   
2
4
2 1
x y z t
r xy yz zt tx xz yt r
  
 
     

Từ 2 bất đẳng thức trên, ta có điều phải chứng minh.
Đẳng thức xảy ra khi và chỉ khi a = b = c= d = r.

Ví dụ 4. Cho a, b, c > 0 thỏa mãn
2 2 2
1
a b c
  
. Chứng minh rằng:



2
2 2 2
3

.
1 1 1 4
a b c
a a b b c c
b c a
    
  

Lời giải: Áp dụng bất đẳng thức Cauchy=Schwarz, ta có:



 
2
3
2 2 2 2
2 2 2
1
cyc cyc
cyc
a a b b c c
a a
b a b a
a b a
 
 
 

 






2
2 2
1
cyc
a a b b c c
a b
 







 
2
2
2 2 2
1
1 .
3
a a b b c c
a b c
 

  



18




2
1
1
3
a a b b c c
 






2
4
3
a a b b c c
  

Suy ra bất đẳng thức cần chứng minh.
Dấu bằng xảy ra khi và chỉ khi
1
3
a b c

  
.
*) Các bạn hãy luyện tập thành thạo bất đẳng thức này với một số bài tập nho nhỏ sau:
1)Cho . Chứng minh rằng:

2) Chứng minh rằng:

3) Chứng minh rằng:

4) Cho . Chứng minh rằng:

5) Cho . Chứng minh rằng:

6) Cho . Chứng minh rằng:

7) Cho . Tìm min, max của
8) Cho . Chứng minh rằng:

19


9) Tìm min, max của
10)Cho . tìm max min của

V) Bất đẳng thức holder
Bất đẳng thức Holder là một bất đẳng thức khá mạnh và có nhiều ứng dụng,
nhưng rất tiếc nó không được phổ biến ở phổ thông hiện nay. Đặc biệt, bất đẳng thức
holder còn được coi là dạng tổng quát của bất đẳng thức AM-GM (với m = 2 ) và cách
chứng minh bất đẳng thức này có sử dụng đến bất đẳng thức AM-GM. Vì thế, chúng tôi
xin giới thiệu bất đẳng thức này trong chuyên đề.

Đầu tiên, xin được trình bày dạng tổng quát của bất đẳng thức :
Với m dãy số dương ( a
1;1
; a
1;2
; … ; a
1;n
) ; ( a
2;1
; a
2;2
; … ; a
2;n
);… ; ( a
m;1
; a
m;2
; … ;
a
m;n
) ta có:

; ;
1 1
1 1
m
m mn n
m
i j i j
j j

i i
a a
 
 
 
 

 
 
 
 
 
 
 

Đẳng thức xảy ra khi m dãy số tương ứng tỉ lệ.
*) Chứng minh:
-Sử dụng bất đẳng thức AM-GM, ta có:

1;1 2;1 ;1
1;1 2;1 ;1
1; 2; ;
1; 2; ;
1 1 1
1 1 1
. .
(1)

m
m

m
n n n
n n n
j j m j
m
j j m j
j j j
j j j
m a a a
a a a
a a a
a a a
  
  
   
    
    
    
  
  


1;2 2;2 ;2
1;2 2;2 ;2
1; 2; ;
;
1 1 1
1
1
. .

(2)
m
m
m
n n n
m
n
j j m j
m
i j
j j j
j
i
m a a a
a a a
a a a
a
  


   
 
 
 
  



………


20


1; 2; ;
1; 2; ;
1; 2; ;
;
1 1 1
1
1
. .
( )
m
n n m n
n n m n
n n n
m
n
j j m j
m
i j
j j j
j
i
m a a a
a a a
n
a a a
a
  



   
 
 
 
  



Cộng từng vế với vế của (1);(2);…;(n), ta được :
;
1
1
; ;
1 1
1 1
;
1
1
.
m
n
m
i j
m m
n n
j
i
m

m
i j i j
m n
j j
i i
m
i j
j
i
m a
m a a
a


 
 


 
  
 
 
 
 
 


 
 





; ;
1 1
1 1
m
m m
n n
m
i j i j
j j
i i
a a
 
 
 
 
 
 
 
 
 
 
 
 

Bất đẳng thức được chứng minh.
Từ bất đẳng thức holder này, ta có một số hệ quả khá quan trọng sau:
Hệ quả 1.

Với a, b, c, x, y, z, m, n, p, là các số thực dương, khi đó, ta có:






 
3
3 3 3 3 3 3 3 3 3
ax
a b c x y z m n p m byn czp
        

Chứng minh.
Thực ra đây chính là hệ quả trực tiếp của bất đẳng thức Holder với m = n = 3.
Sử dụng bất đẳng thức Holder, ta có:
3 3 3
3 3 3 3 3 3 3 3 3
a x m
a b c x y z m n p
 
     

   
3 3 3 3 3 3 3 3 3
3
3axm
a b c x y z m n p


     

Xây dựng tương tự 2 bất đẳng thức (b; y; n) và (c; z; p) rồi cộng theo từng vế, ta
có điều phải chứng minh.

21

Hệ quả 2.
Với dãy số a
1
, a
2
, a
3
,…
,
a
n
ta luôn có:
    


1 2 1 2
1 1 1 1
n
n
n n
a a a a a a
    


Chứng minh.
Sử dụng bất đẳng thức AM-GM, ta có:
    
1 2
1 2
1 1 1

1 1 1
1 1 1
n
n
n
n
a a a
a a a
   
  
  

    
1 2
1 2
1 2
1 2


1 1 1
1 1 1
n
n

n
n
n
n
n a a a
a
a a
a a a
a a a
   
  
  

Cộng từng vế 2 bất đẳng thức trên ta có điều phải chứng minh.
Đây là 2 hệ quả khá quan trọng và sẽ dung nhiều trong công cuộc chinh phục đỉnh
cao bất đẳng thức, vì vậy chúng ta cần nhớ kĩ và vận dung cho thật linh hoạt.
Do chuyên đề tập trung chủ yếu vào hiệu quả của AM-GM nên chúng ta nên dừng
các vấn đề về bất đẳng thức holder tại đây.
Tiếp theo là một số ví dụ có sử dụng bất đẳng thức AM-GM này.

VI) Ứng dụng của bất đẳng thức AM-GM:

Bài 1. Cho a, b, c, là các số thực dương. Chứng minh rằng:
 
2
2 2 2
1 1 1
4
b c c a a b
ab bc ca

a b c a b c
  
   
      
   
   

Lời giải:
Cách 1: Chúng ta có thể viết lại bất đẳng thức cần chứng minh như sau:
   
 
2
2 2 2 2 2 2
4
cyc
ab a b ab bc ca a b b c c a
 
     
 
 

Giả sử
a b c
 
. Khi đó, áp dụng bất đẳng thức AM-GM, ta có:




2 2 2 2 2 2

4
VP ab bc ca a b b c c a
    


22


   


 
2
2 2 2 2 2 2
2
16
4
a b ab bc ca a b b c c a
a b
    




   
 
 
2
2
2 2 2 2 2 2

2
4
4
a b ab bc ca a b b c c a
a b
 
     
 



Bây giờ, ta chỉ cần chứng minh:







2 2 2
ab a b cb c b ac a c
    


   


2
2 2 2 2 2 2
4

a b ab bc ca a b b c c a
a b
     









2 2 2
2 2 2
ab a b c a b c a b
     


  


2 2 2 2 2 2
4
a b b c c a
a b ab bc ca
a b
 
    



     


2 2 2
2 2
2
2
4
4
2 2
c a b
a b
ab a b c a b c a b
a b a b

       
 

 


2 2
2
2
4
a b
ab
ab a b c a b a b
a b a b
 


 
 
       
 
 
 
 
 

 
 
 
2 2
2
2
ab a b c a b
c a b
a b a b
 
   
 

2
2
ab c
c
a b a b
 
 


Điều này hiển nhiên đúng bởi vì:
2 2
2 2
c c ab
c c
a b c c a b
   
  

Bất đẳng thức đã được chứng minh.
Dấu “=” xảy ra khi: a=b=c

Cách 2. Tương tự như cách 1, bất đẳng thức cần chứng minh tương đương :

23

   
 
2
2 2 2 2 2 2
4
cyc
ab a b ab bc ca a b b c c a
 
     
 
 

Ta hãy chú ý đến hằng đẳng thức sau:





     
2 2 2
2 2 2 2 2 2
x y z m n p my nx nz py px mz
         

Bây giờ, ta sẽ áp dụng đẳng thức này với:






, , , , ,
x ab y bc z ca m a b ab n b c bc p a c ac
       

Chúng ta có được:
       
2
2
2
cyc cyc cyc
ab bc ca ab a b ab a b abc c a b
   
      

   
   
  

Hơn nữa, ta lại có:
 


 
2
2
2 2 2 2 2 2
4
cyc cyc
ab a b a b b c c a ab a b
     
 

Vì vậy, ta có thể viết lại bất đẳng thức cần chứng minh như sau:
   
 
2
2 2 2 2 2 2
4
cyc
ab bc ca ab a b a b b c c a
 
      
 
 




     
2
2
cyc cyc
ab bc ca ab a b ab a b
   
    
   
   
 

     
2 2
cyc cyc
ab bc ca ab a b abc c a b
     
 

    
2 2
2 2
0
cyc cyc
a b a b abc a b c a b
      
 


    
2
2 2
2 0
cyc cyc
a b a b abc a a b a c
     
 

Điều này hiển nhiên đúng theo bất đẳng thức Schur bậc 3.

Bài 2. Cho a, b, c, là các số thực dương. Chứng minh rằng:

24

   
2
1
a b c abc
a b b c c a a b b c c a
   
     

Lời giải:
Bất đẳng thức đã cho tương đương với:
 
  
2
2
2

cyc cyc
a ab
a b b c
a b
 
 

 


       
2 2
1 2
abc abc
a b b c c a a b b c c a
 
     

Sử dụng bất đẳng thức quen thuộc sau:

 
   
2
2
2
1
cyc
a abc
a b b c c a
a b

 
  



Ta chỉ cần chứng minh:
          
2 2
2
cyc
ab abc abc
a b b c a b b c c a a b b c c a
 
       


   
2 2 2
2
a b b c c a abc abc a b b c c a
       

Bây giờ, ta giả sử


min , ,
c a b c

, áp dụng bất đẳng thức AM-GM, ta có:





2 2 2 2
a b b c c a abc a ab c bc a b
      






 




2 2
a a b b c a a c b c
bc a b
   
   

  
 
  
 
2 .
2 2
a a b b c a a b b c

bc a b bc a b
   
    


   
2
abc a b b c c a
   

Bất đẳng thức được chứng minh. Dấu “=” xảy ra khi: a = b = c.



25

Bài 3. Cho a, b, c, là các số thực dương. Chứng minh rằng:
 
 
 
 
 
 
2 2 2
3
2 2 2
b c a c a b
a b c a b a c b c a b c
  
  

     

Lời giải:
Cách 1. Sau khi sử dụng bất đẳng thức AM-GM, ta có được bất đẳng thức cần chứng
minh tương đương với:












2 2 2
2 2 2
b c a c a b abc b c a a c b a b c
         

Tiếp tục, áp dụng bất đẳng thức Cauchy-Schwar và bất đẳng thức AM-GM, ta có:
      


2
2 2
b c a c a b b c a bc
     



 
 
2
2
2
a b c
bc b c bc b c a
bc

 
     
 
 

Chứng minh tương tự, ta có:
     
2 2
2
b c a c a b ab a b c
     

      
2 2
2
b c a c a b ca a c b
     

Nhân các bất đẳng thức này với nhau rồi lấy căn, ta được bất đẳng thức cần chứng

minh. Đẳng thức xảy ra khi và chỉ khi: a = b = c.

Cách 2. Ta cần chứng minh bất đẳng thức:
         
2 2 2
2 2 2
b c a c a b abc b c a a c b a b c
         

Theo bất đẳng thức AM-GM, ta có:
     
3
64
2 2 2
27
abc b c a a c b a b c abc a b c
        


   
2 2
64
81
abc a b c ab bc ca
    

Ta sẽ chứng minh:

×